Two ropes in a vertical plane.... sin(theta) or cos(theta)?

In summary: We don't need the angle from the combined vector because it's a combination of two vectors that we already know the positions of and not a separate and unrelated vector. So I used θA=47 and θB=133. Then I think I made a mistake using cos instead of sin. So I have magnitude(A)sin(47)+magnitude(B)sin(133), sub in magnitude(A)=magnitude(B), and use sin instead of cos, and I get magnitude(A)=magnitude(B)=169.
  • #1
Breadsticks
16
1

Homework Statement

:[/B]
Two ropes in a vertical plane exert equal-magnitude forces on a hanging weight but pull with an angle of 86 degrees between them. What pull does each one exert if their resultant pull is 372N directly upward?

Homework Equations

:[/B]
x-component= magnitude*cos(theta)
y-component= magnitude*sin(theta)
Cy=Ay+By

The Attempt at a Solution

:[/B]
Vectors A, B, and resulting vector C are all in the same plane. If they pull equally, we see the angle between A or B and C is 43 degrees. The x-direction forces cancel, so we are left with the y-direction. If we let A=B, we can simplify. If we add vectors A and B together in the x-direction, we get C=Ay+By, and we can equate it with C=372N. So I have 372=2*magnitude*sin(theta). However, the equation the book uses is 372=2*magnitude*cos(theta). Why? Shouldn't it be sin because it's in the y-direction?
 
Physics news on Phys.org
  • #2
Breadsticks said:

Homework Statement

:[/B]
Two ropes in a vertical plane exert equal-magnitude forces on a hanging weight but pull with an angle of 86 degrees between them. What pull does each one exert if their resultant pull is 372N directly upward?

Homework Equations

:[/B]
x-component= magnitude*cos(theta)
y-component= magnitude*sin(theta)
Cy=Ay+By

The Attempt at a Solution

:[/B]
Vectors A, B, and resulting vector C are all in the same plane. If they pull equally, we see the angle between A or B and C is 43 degrees. The x-direction forces cancel, so we are left with the y-direction. If we let A=B, we can simplify. If we add vectors A and B together in the x-direction, we get C=Ay+By, and we can equate it with C=372N. So I have 372=2*magnitude*sin(theta). However, the equation the book uses is 372=2*magnitude*cos(theta). Why? Shouldn't it be sin because it's in the y-direction?

You need a diagram showing where you have put ##\theta## in relation to the ropes.

You're equations above in section 2 are only valid where ##\theta## is the angle with the horizontal.
 
  • Like
Likes Breadsticks
  • #3
PeroK said:
You need a diagram showing where you have put ##\theta## in relation to the ropes.

You're equations above in section 2 are only valid where ##\theta## is the angle with the horizontal.

Ah, so I rotate the x,y axis by 90 degrees and I'm left with cos(theta).

If I wanted to keep the x,y axis in the traditional location, is there an easy way to modify the equation?
 
  • #4
Breadsticks said:
Ah, so I rotate the x,y axis by 90 degrees and I'm left with cos(theta).

If I wanted to keep the x,y axis in the traditional location, is there an easy way to modify the equation?

You'd be better off remembering that sin = opposite/hypotenuse and cos = adjacent/hypotentuse. A triangle, in general, could have any orientation, so its sides will not always be aligned with the x and y axes.
 
  • #5
PeroK said:
You'd be better off remembering that sin = opposite/hypotenuse and cos = adjacent/hypotentuse. A triangle, in general, could have any orientation, so its sides will not always be aligned with the x and y axes.
Okay.

Is there a way to make it work using x-component=magnitude*cos(theta) and y-component=magnitude*sin(theta) if I use two different angles (47 and 133)? I had A use an angle of 133 and B use 47. Then since they cancel each other out, I did magnitude(A)cos(theta)= -magnitude(B)cos(theta) and got 2magnitude(A)=magnitude(B). Then I had magnitude(A)sin(47)+magnitude(B)sin(133), subbed using 2magnitude(A)=magnitude(B) and ended up with magnitude(A)=169N. A would then have a magnitude of 338 and B would have a magnitude of 169. Obviously that's not correct, but I can't tell if it was an assumption or it's just not a viable method.

Edit: This is because it assumes the angle is always the angle from the object it's working on and not just from an axis, right?
 
  • #6
You need to draw a picture

You will find the picture consists of two-right triangles, of exact same dimensions. You know length of the base, you know the angles, calculate the hypotenuese.
 
  • #7
Breadsticks said:
Okay.

Is there a way to make it work using ##\vec V_x= |\vec V| \cos\theta\quad {\rm and} \quad \vec V_y= |\vec V| \sin\theta## if I use two different angles (47 and 133)? ##\quad ##
yes
I had A use an angle of 133 and B use 47. Then since they cancel each other out, I did ##|\vec A| \cos\theta = -|\vec B|\cos\theta## and got ##2 |\vec A| = |\vec B|. \quad ##
wrong twice.

##|\vec A| \cos\theta_A = -|\vec B|\cos\theta_B\Rightarrow |\vec A| = |\vec B|## because ## \cos\theta_A = -\cos\theta_B##, and where the 2 comes from I don't know.

Then I had ##|\vec A|\sin{47^\circ} +|\vec B|\sin33^\circ##, subbed using 2|A|=|B| and ended up with |A|=169 N. A would then have a magnitude of 338 and B would have a magnitude of 169. Obviously that's not correct, but I can't tell if it was an assumption or it's just not a viable method.

Edit: This is because it assumes the angle is always the angle from the object it's working on and not just from an axis, right?[/QUOTE]

Try again. And: You did make a drawing I hope ? Drawings are very, very useful.


AddVecs.jpg
 
  • Like
Likes Breadsticks
  • #8
BvU said:
|A→|cosθA=−|B→|cosθB⇒|A→|=|B→||\vec A| \cos\theta_A = -|\vec B|\cos\theta_B\Rightarrow |\vec A| = |\vec B| because cosθA=−cosθB \cos\theta_A = -\cos\theta_B, and where the 2 comes from I don't know.
I must've mistyped on my calculator because I did cos(47)/-cos(133) and got 2. Calculator memory is cleared so I'm not sure what I really typed in. So using the correct substitution, that the magnitudes are equal, I got the right answer. Thank you so much everyone, you've been really helpful!

BvU said:
Try again.
We don't need the angle from the combined vector because it's a combination of two vectors that we already know the positions of and not a separate and unrelated vector, right?
 
  • #9
Breadsticks said:
We don't need the angle from the combined vector because it's a combination of two vectors that we already know the positions of and not a separate and unrelated vector, right?
In the problem statement you are given that the resultant pull is directly upward. So there is the angle of the sum vector. You use this when you write ##
|\vec A| \cos\theta_A + |\vec B|\cos\theta_B = 0 ##: the 0 is the horizontal component of the resultant.
 
  • #10
BvU said:
In the problem statement you are given that the resultant pull is directly upward. So there is the angle of the sum vector. You use this when you write ##
|\vec A| \cos\theta_A + |\vec B|\cos\theta_B = 0 ##: the 0 is the horizontal component of the resultant.
Okay, thank you.

One more question, in this problem's case, the expression for the x-component would be magnitude*tan(theta), right?
 
  • #11
No. The force points upwards, the ##\theta## you have chosen is wrt the X+ axis ("to the right") so it is ##\pi/2##. Just like with the others, the x component is ##R\cos\theta##, in this case 0.

Try it out with a resultant at e.g. ##\pi/4##
 
  • Like
Likes Breadsticks
  • #12
Breadsticks said:
I must've mistyped on my calculator because I did cos(47)/-cos(133) and got 2. Calculator memory is cleared so I'm not sure what I really typed in. So using the correct substitution, that the magnitudes are equal, I got the right answer.
Coming back to this one. You are supposed to know and use the fact that ##\cos \phi = - \cos (\pi - \phi)##. No calculator should be needed - no typos on that thing can mess it up for you either.

In case you do have to resort to using a calculator: never ever trust the result. So estimate the result any which way you can before you start hammering on the thing.

And the most-often incurred mistake is typing degrees when the setting is for radians (and sometimes vice versa)
 
  • #13
BvU said:
Coming back to this one. You are supposed to know and use the fact that ##\cos \phi = - \cos (\pi - \phi)##. No calculator should be needed - no typos on that thing can mess it up for you either.

In case you do have to resort to using a calculator: never ever trust the result. So estimate the result any which way you can before you start hammering on the thing.

And the most-often incurred mistake is typing degrees when the setting is for radians (and sometimes vice versa)

Yes, thank you.
 
  • Like
Likes erickzind
  • #14
Breadsticks said:
Yes, thank you.

The same question was my homework today. They just got different degrees (°). Thank you for a very nice explanation.
 

What is the difference between sin(theta) and cos(theta)?

Sin(theta) and cos(theta) are both trigonometric functions used to calculate the relationship between the angles and sides of a right triangle. The main difference between them is that sin(theta) calculates the ratio of the opposite side to the hypotenuse, while cos(theta) calculates the ratio of the adjacent side to the hypotenuse.

How do I determine which trigonometric function to use when working with two ropes in a vertical plane?

The choice of trigonometric function depends on the specific information given in the problem. If the known sides of the triangle are the opposite and hypotenuse, use sin(theta). If the known sides are the adjacent and hypotenuse, use cos(theta). If the known sides are the opposite and adjacent, use tan(theta).

Can I use both sin(theta) and cos(theta) in the same problem involving two ropes in a vertical plane?

Yes, it is possible to use both sin(theta) and cos(theta) in the same problem. This may be necessary if the given information involves both the opposite and adjacent sides of the triangle.

What happens if I use the wrong trigonometric function in a problem involving two ropes in a vertical plane?

If you use the wrong trigonometric function, you will get an incorrect answer. It is important to carefully consider the given information and select the appropriate trigonometric function to use in order to accurately solve the problem.

Is there a specific formula for solving problems involving two ropes in a vertical plane?

Yes, the formula used to solve these types of problems is the Pythagorean theorem, which states that in a right triangle, the square of the length of the hypotenuse is equal to the sum of the squares of the lengths of the other two sides. This can be written as a^2 + b^2 = c^2, where c is the length of the hypotenuse and a and b are the lengths of the other two sides. From this formula, we can derive the equations for sin(theta) and cos(theta).

Similar threads

  • Introductory Physics Homework Help
2
Replies
41
Views
2K
  • Introductory Physics Homework Help
Replies
2
Views
630
  • Introductory Physics Homework Help
Replies
2
Views
853
  • Introductory Physics Homework Help
Replies
1
Views
194
  • Introductory Physics Homework Help
Replies
7
Views
1K
  • Introductory Physics Homework Help
2
Replies
44
Views
4K
  • Introductory Physics Homework Help
Replies
1
Views
890
  • Introductory Physics Homework Help
Replies
18
Views
800
Replies
8
Views
232
Back
Top